Reply
 
LinkBack Thread Tools Search this Thread Display Modes
  #1   Report Post  
Old July 21st 03, 05:58 PM
Richard Harrison
 
Posts: n/a
Default basic question about radio waves !!!!

Nic Santeen wrote:
"The RF source applied to a dipole generates a magnetic field and
electric field 90 degrees out of phase (it is a capacitive antenna)."

It could be an inductive antenna or a resonant (resistive) antenna, and
the radiation fields, E&H, would still be at right angles in space, in
the plane of the plane wave, perpendicular to each other and to the
direction of travel.

Static fields distant from a current-carryinng wire are in a small way
analogous. Imagine the current flowing toward you. Electric lines
radiate like spokes radially positioned around the wire. Magnetic lines
are concentric and make circles around the wire. At great diatance from
the wire, the magnetic and electric lines are practically perpendicular
and they are both perpendiicular to the direction of energy flow. Don`t
make too much of this analogy, but it may be illustrative.

Borrow a copy of Terman`s 1955 edition from a library and study page No.
1.

BNest regards, Richard Harrison, KB5WZI

  #2   Report Post  
Old July 21st 03, 07:54 PM
Nic. Santean
 
Posts: n/a
Default

"Richard Harrison" wrote
It could be an inductive antenna or a resonant (resistive) antenna, and
the radiation fields, E&H, would still be at right angles in space, in
the plane of the plane wave, perpendicular to each other and to the
direction of travel.


I am aware of how these vectorial fields look like ...

When I said "magnetic field and electric field are 90 degrees out of
phase (it is a capacitive antenna)", I was refering that in any point in
space, when the electric field peaks, the magnetic field hits zero; when
the electric field drops to zero, the magnetic field change direction and
peaks again in the other direction, etceteras. I said nothing about the
spatial position of their vectors - which is a known, even by me.
I just said that there is a shift(!) between the sinusoidals.

My point is that a clean EM radiation has both fields in phase, which
means that the scalar values of both vectors (electric and magnetic)
change synchronously: when the electric field peaks, the magnetic field
peaks as well, when the electric field hits zero, so does the magnetic
field.

Therefore, I was saying that I suspect the superposition of two different
EM radiation coming out from a dipole. These two fields do not necessarily
reinforce eachother, but rather they create their own counterparts with
wich they go along.

You must have misunderstood me.

Borrow a copy of Terman`s 1955 edition from a library and study page No.

1.

Thank you for the reference.

Cordially,

Nic.


  #3   Report Post  
Old July 22nd 03, 11:38 AM
W5DXP
 
Posts: n/a
Default

OK1SIP wrote:
this a simple, intuitive and maybe completely wrong approach:
Let's consider a travelling EM wave at any point, distant from
antenna. Both the E and H fields change in time. IMHO the power
density at that point should be constant in time.


From _Optics_, by Hecht: Equation 3.43 is the instantaneous flow of
energy per unit area per unit time, i.e. the instantaneous Poynting
Vector and is a cosine squared function. "It should be evident that
E^xH^ cycles from maxima to minima. At optical frequencies, S^ is an
extremely rapidly varying function of time ... Therefore, its
instantaneous value would be an impractical quantity to measure
directly. This suggests that we employ an averaging process."

The average power density (irradiance) is constant, not the
instantaneous power density which is a cosine squared function.

Why should they change "synchronously", causing the power density at
any point swing from zero to "maximum squared" ?


From _Optics_, by Hecht:
"Since Ey and Bz differ only by a scalar, and so have the same time-
dependence, E and B are *in-phase* at all points in space".
--
73, Cecil http://www.qsl.net/w5dxp



-----= Posted via Newsfeeds.Com, Uncensored Usenet News =-----
http://www.newsfeeds.com - The #1 Newsgroup Service in the World!
-----== Over 80,000 Newsgroups - 16 Different Servers! =-----
  #4   Report Post  
Old July 22nd 03, 06:03 PM
Roy Lewallen
 
Posts: n/a
Default

I never saw the original posting, but it's been quoted enough that I
think I understand the original question.

The E and H fields might very well be out of time phase near the
antenna. And very close to the antenna (within a pretty small fraction
of a wavelength), the magnitude of the E/H ratio will be considerably
greater than 377 ohms if the antenna is much shorter than a half
wavelength. However, beyond a wavelength or so, the E/H ratio will be
very close to 377 ohms, and E and H will be in time phase. They'll be
oriented at a 90 degree angle with respect to each other in space, but
will be in time phase.

In the far field, the ratio of E to H (both magnitude and phase) are
dictated by the medium, not the antenna.

Anyone interested in investigating this can do so very easily with
EZNEC, even the demo version, using the near field analysis. It gives
both magnitude and phase of E and H fields at any point in space for any
antenna you can model.

Roy Lewallen, W7EL

  #5   Report Post  
Old July 23rd 03, 04:42 PM
W5DXP
 
Posts: n/a
Default

Richard Harrison wrote:
For the foregoing reasons, it is wrong to say a single radio frequency
has anything to do with "two different EM radiations coming out from a
dipole".


Richard, I think maybe he was referring to the MOM method of dividing a
wire up into 'n' segments and assuming each segment is a small individual
radiator. The radiation from each small radiator is vectorially summed
with the radiation from all the other small radiators. Close to the
radiators, the E and H fields can sum to being out of phase.

If you change his "two different EM radiations" to "N different EM
radiations", then what he is saying might be more understandable.
--
73, Cecil http://www.qsl.net/w5dxp



-----= Posted via Newsfeeds.Com, Uncensored Usenet News =-----
http://www.newsfeeds.com - The #1 Newsgroup Service in the World!
-----== Over 80,000 Newsgroups - 16 Different Servers! =-----


  #6   Report Post  
Old July 23rd 03, 05:45 PM
Nic. Santean
 
Posts: n/a
Default

As for "twin EM signals, 90 degrees out of phase", the right-angle
relationship is spatial, in the plane of the wavefront, and it is not
between twin signals. It is between the lines of the E and M fields.


It appears that I am having difficulties explaining this
"out of phase" business. Let me try once mo

Take a snapshot of the dipole when the charges are crowded at
the ends of the dipole: all negative charges are at one end and
all positive charges are at the other end. At this very moment,
in the proximity of the antenna there is a maximum electric field
with vectors oriented from the "positive" half of the dipole to the
"negative" half. This electric field has just reached its peak. But
in the same time there is close to zero flow of charges in the
antenna, implying that the magnetic field is close to zero.

Indeed, the magnetic field is perpendicular to the electric field;
however, it is zero exactly when the electric field is maximal.
This is called a "90 degree out of (time-)phase".

However - in theory - an accelerated point of charge creates
an EM front with an electric field and a magnetic field in
(time-)phase, i.e. they peak and die together, perpendicular
on each other. Moreover, in free space a varying electric field
generates a magnetic field in (time-)phase with the originating
electric field (without the need of any charge).

So, the problem is that the dipole generates two fields (E&M)
out of (time-)phase; whereas EM propagation in free space
implies two fields in phase. The "contradiction" can not be more
clear than this.

In order to explain this wonder, I was suggesting the following
scenario:

The dipole give us these two ortogonal fields, E1 and M1 : an electric
field and a magnetic field out of time-phase. E1 is a time varying electric
field, hence it creates its own companion - say - M2 which is a magnetic
field in time-phase! with E1. In the same time, M1 is a time-varying
magnetic field, hence it creates its own companion - say - E2 which
is an electric field in time-phase with M1.

Doing the arithmetic, we conclude that we have four fields altogether,
which represent two separate EM emissions: E1&M2 is one emission,
and E2&M1 is the other one. These two emission are obviously out
of time-phase(they inherited from their originators), i.e. when
[E1 and M2] both peak, [E2 and M1] both die. This is like saying that
the two EM radiations are 90 degrees out of time-phase.

From here on, I do not know how to continue. One way may be that
these two EM radiations - as any behaving waves - sum up vectorially,
and lead to a unique EM radiation with an amplitude a bit higher that the
initial ones. The problem I am having with this explanation is that the
wavelength is also a bit shorter - which does not sound right.

So, I am looking for a better continuation - or for a better explanation
of the following : how comes that we start with two out of time-phase E
and M fields and end up with an EM propagation, i.e. with E and M
in time phase ?

Cordially,

Nic.


  #7   Report Post  
Old July 23rd 03, 06:09 PM
Richard Harrison
 
Posts: n/a
Default

Cecil, W5DXP wrote:
"I think maybe he was referring to the MOM method of dividing a wire up
into "n" segments and assuming each segment is a small individual
radiator."

It`s possible. The method of moments (MOM) is one way I`m accustomed to
solving civil and mechanical problems. I learned from the newsgroup that
antenna modelers had appropriated the term when I explained the
mechanical process on-line.

Best regards, Richard Harrison, KB5WZI

  #8   Report Post  
Old July 23rd 03, 06:18 PM
Nic. Santean
 
Posts: n/a
Default


I did the computations : the composition of those
two "EM emissions" 90 degree out of phase (but
having the same period) leads to a wave of exactly
the same wavelength as the signal applied to the
antenna.

So, there is no contradiction in my scenario so far.

Nic.
http://www.csd.uwo.ca/~nic


  #9   Report Post  
Old July 23rd 03, 07:13 PM
W5DXP
 
Posts: n/a
Default

Nic. Santean wrote:
So, the problem is that the dipole generates two fields (E&M)
out of (time-)phase; whereas EM propagation in free space
implies two fields in phase. The "contradiction" can not be more
clear than this.


If the antenna is a traveling-wave antenna, the E-fields and H-fields will
be in time phase, physically orthogonal to each other's plane and orthogonal
to the direction of energy flow. Reference: "Antennas", Kraus & Marhefka,
third edition, sec. 6-8. "Since the current is entirely in the 'z' direction,
the magnetic field has but ONE component, H(epsilon)." The ratio of the E-
field to the H-field will be the characteristic impedance of the traveling-
wave antenna. Also: _Optics_, by Hecht, Section 3.2, Electromagnetic Waves.

For standing-wave antennas, like a dipole, there is MORE THAN ONE magnetic
field component.

For standing-wave antennas, like a dipole, there are forward-traveling
waves from the feedpoint to the ends of the antenna and reflected
traveling waves reflected back from the ends. This is similar to an un-
terminated transmission line. It is well known that the total net voltage
and total net current can be 90 degrees out of phase in a lossless un-
terminated transmission line. Approximately the same holds true for
standing-wave antennas. The total net voltage is the phasor sum of
the forward voltage and reflected voltage which relates to the total
E-field. The total net current is the phasor sum of the forward current
and reflected current which relates to the total H-field. And of
course, as in a transmission line, the total voltage and total current
can be almost 90 degrees out of phase at the two points equi-distant from
the feedpoint and the ends. So of course, the E-field and the H-field near
the antenna will possess the same phase as the total net voltage and total
net current flowing in the wire at the same orthogonal plane.
--
73, Cecil http://www.qsl.net/w5dxp



-----= Posted via Newsfeeds.Com, Uncensored Usenet News =-----
http://www.newsfeeds.com - The #1 Newsgroup Service in the World!
-----== Over 80,000 Newsgroups - 16 Different Servers! =-----
  #10   Report Post  
Old July 23rd 03, 07:17 PM
W5DXP
 
Posts: n/a
Default

Richard Harrison wrote:

Cecil, W5DXP wrote:
"I think maybe he was referring to the MOM method of dividing a wire up
into "n" segments and assuming each segment is a small individual
radiator."

It`s possible. The method of moments (MOM) is one way I`m accustomed to
solving civil and mechanical problems. I learned from the newsgroup that
antenna modelers had appropriated the term when I explained the
mechanical process on-line.


Or, based on further thought, he may be talking about the forward-traveling
wave plus the reflected traveling wave on a standing wave antenna. Please
see my other posting. The fields generated by those two traveling-waves
are superposed upon each other in the vicinity of a standing-wave wire
antenna and can certainly put them out of time phase with each other.
--
73, Cecil http://www.qsl.net/w5dxp



-----= Posted via Newsfeeds.Com, Uncensored Usenet News =-----
http://www.newsfeeds.com - The #1 Newsgroup Service in the World!
-----== Over 80,000 Newsgroups - 16 Different Servers! =-----
Reply
Thread Tools Search this Thread
Search this Thread:

Advanced Search
Display Modes

Posting Rules

Smilies are On
[IMG] code is On
HTML code is Off
Trackbacks are On
Pingbacks are On
Refbacks are On


Similar Threads
Thread Thread Starter Forum Replies Last Post
basic question about radio waves Nic. Santean Antenna 14 July 19th 03 06:47 PM
Amateur Radio Legal Issues List Amateur Radio Station N0JAA Antenna 0 July 19th 03 04:30 AM


All times are GMT +1. The time now is 06:50 AM.

Powered by vBulletin® Copyright ©2000 - 2024, Jelsoft Enterprises Ltd.
Copyright ©2004-2024 RadioBanter.
The comments are property of their posters.
 

About Us

"It's about Radio"

 

Copyright © 2017